Retirement Mocktest

17
Financial Planning Academy Retirement Planning Mock Test: Practical Questions Section 1: 29 Questions (1 Mark each): 1) An establishment with 100 employees, notified by Central Government and also that which is a factory engaged in a specified industry, would be covered under Employees Provident fund and Miscellaneous provision Act, 1952 if the number of employees, getting salary of Rs.6500 or less, is/are _____________. A) 20 or more B) It will be covered even if the income of all employees is above Rs. 6500 C) 1 or more D) More than 20 2) Which of the following is a defined contribution plan? A) Leave salary B) Gratuity C) Unrecognized Provident Fund D) Voluntary Retirement Scheme 3) As an employee Rajesh has come to you with his questions on superannuation plans/ Annuity Plans to get more educated with current scenario. Deduction in respect of contribution for annuity plan to certain pension fund under 80C is allowed to a/an __________? A) Individual resident in Indian B) Individual Assessee only C) Individual or HUF D) Any Assessee 4) As an employee Mohan has come to you with his question on superannuation plans/ Annuity Plans to get more educated with current scenario. The term 'Employee benefits' includes contributions in which of the following categories of schemes? . A) Non occupation disability Insurance B) Unemployment compensation Insurance C) Social security Scheme D) Group Insurance Scheme 5) To what extent the employer.s contribution to the PF and the Superannuation Schemes is treated as deductible expense? . A) 12% of the salary of the employee B) 15% of the salary of the employee C) 24% of the salary of the employee D) 27% of the salary of the employee

Transcript of Retirement Mocktest

Page 1: Retirement Mocktest

Financial Planning Academy

Retirement Planning Mock Test: Practical Questions

Section 1: 29 Questions (1 Mark each):

1) An establishment with 100 employees, notified by Central Government and also that

which is a factory engaged in a specified industry, would be covered under Employees

Provident fund and Miscellaneous provision Act, 1952 if the number of employees,

getting salary of Rs.6500 or less, is/are _____________.

A) 20 or more

B) It will be covered even if the income of all employees is above Rs. 6500

C) 1 or more

D) More than 20

2) Which of the following is a defined contribution plan?

A) Leave salary

B) Gratuity

C) Unrecognized Provident Fund

D) Voluntary Retirement Scheme

3) As an employee Rajesh has come to you with his questions on superannuation plans/

Annuity Plans to get more educated with current scenario. Deduction in respect of

contribution for annuity plan to certain pension fund under 80C is allowed to a/an

__________?

A) Individual resident in Indian

B) Individual Assessee only

C) Individual or HUF

D) Any Assessee

4) As an employee Mohan has come to you with his question on superannuation plans/

Annuity Plans to get more educated with current scenario. The term 'Employee benefits'

includes contributions in which of the following categories of schemes? .

A) Non occupation disability Insurance

B) Unemployment compensation Insurance

C) Social security Scheme

D) Group Insurance Scheme

5) To what extent the employer.s contribution to the PF and the Superannuation Schemes

is treated as deductible expense? .

A) 12% of the salary of the employee

B) 15% of the salary of the employee

C) 24% of the salary of the employee

D) 27% of the salary of the employee

Page 2: Retirement Mocktest

Financial Planning Academy

6) Deferred Annuity means ___________. .

A) purchase price is paid in a single installment

B) annuity payments begin at the end of a selected term

C) purchase price is paid in regular installments spread through the period of deferment

D) All of the above.

7) Under EPF scheme, monthly pension after retirement is decided on _________. .

A) eligible service and pensionable salary

B) pensionable service and pensionable salary

C) actual service and pensionable salary

D) None of the above.

8) The maximum advance a member can take from his/her Provident Fund, for major

illness involving stay in hospital for at least 1 month, is ______. .

A) 12 months wage (basic + DA)

B) 24 months wage (basic + DA)

C) 6 months wage (basic + DA)

D) 36 months wage (basic + DA)

9) Sundar has been an employee of a public sector undertaking for the past 25 years and

is retiring the next year. He is eligible for gratuity as per the provisions of the Payment of

Gratuity Act, 1972. He hopes to invest the proceeds along with the PF proceeds, in order

to fund his retirement. Sundar does not receive his gratuity 45 days after his retirement.

What are his rights?

A Sundar should receive his gratuity within 30 days after his retirement. The employer

has to pay penal interest for delay in the payment of gratuity.

B Sundar should receive his gratuity within 60 days after his retirement. He should wait

for his settlement.

C Sundar should receive his gratuity within 21 days after his retirement. The employer

has to pay penal interest at 15% for delay in the payment of gratuity.

D There is no legally determined time frame for payment of gratuity. Sundar has legal

remedies if payment is not made "within reasonable time."

10) Amit is eligible for gratuity as per the provisions of the Payment of Gratuity Act,

1972. Amit’s employer has agreed to pay him a gratuity amount that is higher than what

is stipulated under the Act. Amit accordingly would receive a final settlement of Rs.

5,00,000/- on retirement. What is the tax status of this amount?

A The gratuity paid is exempt from Income Tax only to the extent of Rs. 3,50,000/-.

B The amount of gratuity payable to him cannot exceed Rs. 3,50,000/-.

C There is no specific tax exemption for gratuity, in the Income Tax Act.

D The Income Tax Act will only allow a maximum exemption upto 15 day’s wages per

completed year of service. The rest is taxable.

Page 3: Retirement Mocktest

Financial Planning Academy

11) Deposit Insurance and Credit Guarantee Corporation insures deposits in Banks up

to the following limit per account.

A Rs. 0.5 lakh.

B Rs. 1.0 lakh.

C Rs. 1.5 lakh.

D No insurance is available.

12) An eligibility criterion for withdrawal from a Provident fund for purchase of a house

is ______________.

A) membership of the fund for 5 years, minimum balance in members a/c Rs. 1,000,

purchase should be in favour of the member or member and spouse.

B) membership of the fund for 10 years, minimum balance in members a/c Rs. 1,000,

purchase should be in favour of the member or member and spouse.

C) membership of the fund for 5 years, minimum balance in members a/c Rs. 5,000,

purchase

should be in favour of the member or member and spouse.

D) membership of the fund for 5 years, minimum balance in members a/c Rs. 1,000,

purchase should be in favour of the member or spouse.

13) Which amongst the following is true about the interest received from Recognised

Provident Fund (RPF)?

A) Not treated as income in the year of credit.

B) Not treated as income if the rate is equal to/less than 8.5%. Rate over and above this is

taxable

C) Fully exempt from tax.

D) None of the above

14) Investment in Senior Citizens Pension Plan qualifies for deduction u/s 80C of IT Act.

A) True

B) False

C) Only upto Rs. 70,000

D) Data insufficient

15) Which one of the following statements is NOT true for a defined-benefit plan.

(i) It favors older participants.

(ii) Contributions are annual & arbitrary.

(iii) It requires an actuary. .

A) (i) only

B) (ii) only

C) (iii) only

D) None of the statements

Page 4: Retirement Mocktest

Financial Planning Academy

16) Senior Citizens Savings Scheme is governed by which Act?

A) Small Savings & Misc. Savings Act, 1902

B) Banking Regulation Act, 1949

C) Government Savings Bank Act, 1873

D) Companies Act, 1956

17) (i) A fixed benefit annuity is an annuity under which the insurer guaranties that at

least a defined

amount of annuity benefit will be provided for each rupee applied to purchasing the

annuity.

(ii) A fixed benefit annuity is an annuity under which the insurer guaranties that a fixed

amount of annuity benefit will be provided for life.

A) Only (i) is correct.

B) Only (ii) is correct.

C) Both (i) & (ii) are correct

D) Both (i) & (i) are wrong.

18) A PPF account was opened on 10th July 1990. It will mature on ________.

A) 1/4/2006

B) 9/7/2005

C) 10/7/2005

D) 11/7/2005

19) Deficit financing provision in the Budget will lead to the following/s.

(A) Increase in money supply

(B) Adjustment of interest rate

(C) Rise in prices

(D) Fall in national income.

A) (A) only

B) (A) & (C) only.

C) (A), (B) & (C) only

D) All the above

20) Savita's monthly contribution to her company Provident Fund exceeds the statutory

requirement by Rs. 5,000. She plans to visit her son in the United States soon after

retirement. If she does not claim her funds soon after retirement, what are her rights?

A) The fund will accumulate interest for a period of six months, beginning one month

after retirement. No interest is payable after this period.

B) The fund will continue to accumulate interest and she can claim it any time after her

return.

C) The fund will accumulate interest for one year, from the date of retirement.

D) The fund will stop accumulating interest from the date of her retirement.

Page 5: Retirement Mocktest

Financial Planning Academy

21) The process of retirement planning would generally not involve ________.

A) concentrating on maximizing returns on Present / Past Investments

B) projecting individual needs and goals into the future and making sound financial plan

C) making a plan for management & disposition of assets at death

D) planning for staying physically healthy & making necessary psychological adjustment

and to plan for housing

and leisure / work

22) The matching contribution by the Government, to the Contributory Provident Fund,

for its employees, is ____________.

A) 8.33%

B) 12%

C) 10%

D) None of the above

23) For defined benefit plans, which of the following changes in actuarial assumption

would increase plan costs to an employee?

1. Early retirement without a reduction in benefits.

2. Longer life expectancy predicted

3. An increase in inflation expectations regarding labor costs.

A 1 only

B 1 & 2 only

C 2 & 3 only

D 1, 2 & 3

24) Savita has been working and is covered by the Provident Fund Act, 1952. She hopes

that the savings in the scheme would be adequate to fund her retirement. She would like

to enhance her contribution and make an additional monthly contribution of Rs. 5000.

Can Savita take an advance from her PF account for funding her medical expenses?

A Yes. She can take an advance provided she can repay it within 5 years.

B No, she cannot take an advance for medical expenses. These have to be covered by

insurance.

C Yes, she can take all of her voluntary contribution and upto three-fourths of her total

contribution.

D Yes, she can take an advance of half of the amount in her credit, or 6-months pay,

which ever is less.

Page 6: Retirement Mocktest

Financial Planning Academy

25) Sundar has been an employee of a public sector undertaking for the past 25 years and

is retiring the next year. He is eligible for gratuity as per the provisions of the Payment of

Gratuity Act, 1972. In the event of Sundar.s untimely death before retirement, what

happens to the Gratuity payable to him?

A Gratuity is payable to his nominee, or his heirs as the case may be.

B Gratuity is not payable if the employee dies before retirement from service.

C Gratuity is payable only if the employee has 5 years of continuous service as defined

by the Act.

D Gratuity is paid at a lower rate.

26) Which of the following is FALSE about Gratuity as per Payment of Gratuity Act

1972?

A Gratuity is payable only in the case of retirement or superannuation, not in the case of

resignation or termination of service.

B The amount of gratuity payable cannot exceed Rs. 350000.

C The terms of gratuity can be better than what is stipulated in the Act.

D Gratuity payable can be forfeited by the employer for specific offences committed

during employment.

27) As an employee Rajesh has come to you with his questions on superannuation plans/

Annuity Plans to get more educated with current scenario. What is the maximum benefit

allowed under Employees Deposit linked Insurance?

A Average Balance in the provident fund account during the past 12 months or during

the period of membership whichever is less.

B 25000+40% of the amount in excess of Rs. 25000.

C 25000+25% of the amount in excess of Rs. 25000.

D Rs. 60000.

28) Pension received from a former employer is classified under the head _________.

A) Gratuity

B) Income from other sources

C) Salary

D) Capital Gains

29) For a person aged 25 years retirement needs may best be calculated by

_____________.

A) Income replacement method

B) Expense replacement method

C) Both

D) None of the above.

Page 7: Retirement Mocktest

Financial Planning Academy

Section 2: 23 Questions (2 Marks each):

1) Sushil has an accumulated amount of Rs 75 lakh at the time of his retirement. Also,

just before retirement, his household expenses are Rs. 2.40 lakh pa and he wishes to

maintain the same standard of living after retirement. If inflation is 6.5% p.a. and interest

on investment is 9% pa, how long would this money last if he gives Rs. 25 lakh estate to

his son out of the accumulated amount? .

A) 31.25 Years

B) 30 Years

C) 23 Years

D) None of the above

2) Shyam opens his PPF account in the year 2001- 02. When can he make his first

withdrawal from this PPF account? .

A) After 31-03-2006

B) After 31-03-2008

C) After 31-03-2007

D) After 31-03-2016

3) Manish is working in a Private Ltd company covered under PF Act. His salary is Rs.

8500 pm (basic Rs. 6500 + DA Rs. 2000). His employer contributes towards employee

benefits upto the specified salary limit. What would be the contribution of the employer

towards (A) EPS, (B) EPF for Manish?

A) Rs. 541, Rs. 239

B) Rs. 239, Rs. 541

C) Rs. 780, Rs. 239

D) None of the above.

4) Amit works as a seasonal employee in an establishment covered under Gratuity Act.

His date of joining is 01 Apr 1977, and works till 31 Mar 2007. His wages (Basic + DA)

at the time of retirement is Rs. 10000 per month. Work out his gratuity entitlement. .

A) Rs. 115385

B) Rs. 80769

C) Rs. 173077

D) Rs. 350000

5) Brijesh, age 48, plans to retire at 65 and wants to be debt free at retirement. The

balance sheet mortgage is Rs. 114042 at the end of the 10th year of a 30 year loan. The

monthly payment was Rs. 953.89. What was the original balance of the loan if the

interest rate was 8%? (Select closest answer)

A) Rs. 140000

B) Rs. 130000

C) Rs. 120000

D) Rs. 125000

Page 8: Retirement Mocktest

Financial Planning Academy

6) Gary received an inheritance of Rs. 2 Lakh. He wants to withdraw equal periodic

payments at the beginning of each month for 5 years starting after 5 years. He expects to

earn 12% annual interest, compounded monthly on his investments. How much can he

receive each month? .

A) Rs. 8082.28

B) Rs. 4448.89

C) Rs. 4404.84

D) Rs. 8002.26

7) The maximum amount payable under group gratuity is _________.

A Rs. 3 lakh

B Rs. 3.50 lakh

C Rs. 2.5 lakh

D 5% of salary

8) Ajay is a small scale trader in nuts and has been making a modest profit for the past 12

years. He has pre-dominantly used the PPF to save for his future. He also has a welfare

scheme for his few employees, for whom he runs a small provident fund, which is yet to

be recognised. He also contributes to this fund as an employer and matches the

employee's contribution. He has encouraged his employees also to open PPF accounts to

save tax and set aside funds for the future. Ajay has called you to take a look at what he

has been doing and advise him on the choice of products for providing retirement benefits

to himself and his employees. Which of the following describes the status of the actual

retirement benefits received by Ajay's employees from the unrecognised PF account, on

retirement?

A The employee's own contribution will be fully tax exempt. The employer's

contribution and interest are taxable under the head 'salary'.

B The entire proceeds are exempt from tax.

C The entire proceeds are subject to tax.

D The employee and employers. contribution are exempt. Interest earned is taxable.

9) Which of the following statement is correct?

A Payment under a deferred Annuity Plan starts on retirement

B Payment under a deferred Annuity Plan starts after a specified term

C Payment under a deferred Annuity Plan starts from the age 55 years

D All the above

10) Mr. Gaurav is working with APE Ltd. since October 1, 1993. He is entitled for the

basic salary of Rs.6000 per month and Dearness Allowance is 40% of Basic Salary for

retirement benefits. He retired from his job on January 1, 2007 (3 months before the end

of F.Y 2007) and shifted to his village. He is entitled to the following benefits at the time

of retirement :- Gratuity = Rs.98000. Pension from January 1, 2007 = Rs. 2000 per

month. Payment from recognized PF = Rs. 300000. Encashment of earned leave for 150

days = Rs. 36000. He was entitled to 40 days leave for every completed year of service.

Page 9: Retirement Mocktest

Financial Planning Academy

He got 50% of his pension commuted in lump sum w.e.f March 1, 2007 and received Rs.

120000 as commuted pension. Gaurav contributes Rs. 900 per month to RPF to which his

employer contributes an equal amount. With these details you are required to help him

with the following questions related to previous year 2006-07. What amount will he get

as the tax exemption under the Payment of Gratuity Act, 1972?

A Rs. 63,000/-

B Rs. 67,846/-

C Rs. 35,000/-

D Rs. 3,50,000/-

11) Which of the following statement is correct?

A Actuary uses Mortality Table to fix up Annuity Premium

B Annuitants live longer than most persons

C Actuary uses special Annuity Table to calculate Annuity Premium

D All the above

12) Vishal is working with Amex Ltd since October 1, 1993. He is entitled to a basic

salary of Rs. 6,000 per month. Dearness Allowance is 40% of Basic Salary for retirement

benefits. He retired from his job on January 1, 2007. He is entitled to the following

benefits at the time of retirement. Gratuity = Rs. 98,000. Pension from January 1, 2007 =

Rs. 2,000 per month.

Payment from recognized PF = Rs. 3,00,000. Encashment of earned leave for 150 days =

Rs. 36,000. He was entitled to 40 days leave for every completed year of service. He got

50% of his pension commuted in lump sum w.e.f April 1, 2007 and received Rs. 1,20,000

as commuted pension. Vishal contributes Rs. 900 per month to RPF to which his

employer contributes an equal amount. What will be the amount of un-commuted pension

for Vishal that will form part of his total income for the A.Y. 2007-08?

A) NIL.

B) Rs. 4,000

C) Rs. 6,000

D) Rs. 1,000

13) A Provident Fund acquires its status of recognition by ___________.

A) Labour Ministry

B) SEBI

C) PF Commissioner

D) IT Authority

14) Mr. Prasad is a small scale trader. He has a welfare scheme for his few employees,

for whom he runs a small provident fund, which is yet to be recognised. He also

contributes to this fund as an employer and matches the employees. contribution. He has

encouraged his employees to open PPF accounts to save tax and set aside funds for the

future. Can Mr. Prasad open joint PPF accounts with his employees?

A) Yes, it can be opened, but the proceeds cannot be made payable to either or survivor.

B) A PPF account cannot be opened in joint names.

Page 10: Retirement Mocktest

Financial Planning Academy

C) No, joint accounts can be held only by members of the same family or HUFs.

D) Yes, provided the employee is the first holder.

15) Raj joins ABC Ltd. on 25th April 1967. ABC has 30 employees. If he is expected to

retire on 15th November 2007, and his last drawn salary for gratuity purposes is Rs.

26,000 per month, what would be ABC Ltd.s liability on account of Raj.s gratuity

entitlement if ABC Ltd is covered under the payment of Gratuity Act, 1972? .

A) Rs. 6,00,000.

B) Rs. 5,20,000.

C) Rs. 3,50,000.

D) Rs. 3,12,000.

16) Kulkarni is running an unrecognised Provident Fund, because he has a smaller

number of employees in his firm. Which of the following is true about the tax status of

such a fund?

A) Only the employees contribution is eligible for a rebate under Section 80C.

B) Kulkarni’s contribution as the employer is exempt from tax in his books.

C) The interest received on such unrecognised PFs is taxable every year.

D) Accumulated employee.s contribution is taxable at the time of retirement.

17) Abhishek is working for the Finance Ministry since October 1, 1993. He is entitled to

a basic salary of Rs. 6,000 per month. Dearness Allowance is 40% of Basic Salary for

retirement benefits. He retired from his job on January 1, 2007. He is entitled to the

following benefits at the time of retirement. Gratuity = Rs. 98,000. Pension from January

1, 2007 = Rs. 2,000 per month. Payment from recognized PF = Rs. 3,00,000. Encashment

of earned leave for 150 days = Rs. 36,000. He was entitled to 40 days leave for every

completed year of service. He got 50% of his pension commuted in lump sum w.e.f

March 1, 2007 and received Rs. 1,20,000 as commuted pension. Abhishek contributes Rs.

900 per month to RPF to which his employer contributes an equal amount. What is the

amount of commuted pension that will be exempt for Abhishek for the FY 2006-07?

A) Rs. 1,20,000

B) Rs. 80,000

C) Rs. 40,000

D) NIL

Page 11: Retirement Mocktest

Financial Planning Academy

18) Mr. Kalpesh decides to accumulate Rs. 50 lakh when he retires. He is 30 years old at

present and wants to retire at the age of 55 years. Assume Interest Rate = 9%, Inflation =

5%. Compounding to be done on annuity certain basis. If Mr. Kalpesh could save only

Rs.45, 000 p.a. for first ten years, how much does he need to save for next 15 years to

meet his retirement nest egg?

A Rs. 147008 p.a

B Rs. 98450 p.a

C Rs. 79250 p.a

D Rs. 85477 p.a

19) Mr. Gaurav is working with APE Ltd. since October 1, 1992. He is entitled for the

basic salary of Rs.6000 per month and Dearness Allowance is 40% of Basic Salary for

retirement benefits. He retired from his job on January 1, 2006 (3 months before the end

of F.Y 2006) and shifted to his village. He is entitled to the following benefits at the time

of retirement. Gratuity = Rs. 98000. Pension from January 1, 2005 = Rs. 2000 per month.

Payment from recognized PF = Rs. 300000. Encashment of earned leave for 150 days =

Rs. 36000. He was entitled to 40 days leave for every completed year of service. He got

50% of his pension commuted in lump sum w.e.f March 1, 2006 and received Rs. 120000

as commuted pension. Gaurav contributes Rs. 900 per month to RPF to which his

employer contributes an equal amount. With these details you are required to help him

with the following questions related to Assessment year 2006-07. What is the amount of

leave encashment he is eligible for exemption?

A Nil

B 10 month.s average salary

C Actual Encashed salary

D None of the above

20) Mr. Gaurav is working for the finance ministry since October 1, 1991. He is entitled

for the basic salary of Rs.6000 per month and Dearness Allowance is 40% of Basic

Salary for retirement benefits. He retired from his job on January 1, 2005 (3 months

before the end of F.Y 2005) and shifted to his village. He is entitled to the following

benefits at the time of retirement. Gratuity = Rs.98000. Pension from January 1, 2005 =

Rs. 2000 per month. Payment from recognized PF = Rs. 300000. Encashment of earned

leave for 150 days = Rs. 36000. He was entitled to 40 days leave for every completed

year of service. He got 50% of his pension commuted in lump sum w.e.f March 1, 2005,

and received Rs. 120000 as commuted pension. Gaurav contributes Rs. 900 per month to

RPF to which his employer contributes an equal amount. What is the amount of

commuted pension that will be exempted?

A NIL

B Rs. 40000

C Rs. 80000

D Rs. 120000

Page 12: Retirement Mocktest

Financial Planning Academy

21) A recognized Provident Fund acquires its status on recognition by ___________.

A) SEBI

B) Income Tax authorities

C) PF Commissioner

D) Labour Ministry

22) (A) Interest income of the Employees. contribution in an Unrecognised Provident

fund is Tax exempt.

(B) The employees contribution to an unrecognised Provident fund is tax exempt under

Sec. 80 C upto the limit of Rs. 1,00,000.

A) (A) only is correct

B) (A) & (B) both are wrong.

C) (A) & (B) both are correct

D) (B) only is correct.

23) Abhishek is working for the Finance Ministry since October 1, 1993. He is entitled to

a basic salary of Rs. 6,000 pm. Dearness Allowance is 40% of Basic Salary for retirement

benefits. He retired from his job on January 1, 2007 (3 months before the end of F.Y

2006-07 ) and shifted to his village. He is entitled to the following benefits at the time of

retirement. Gratuity = Rs. 98,000. Pension from January 1, 2007 = Rs. 2000 per month.

Payment from recognized PF = Rs. 3,00,000. Encashment of earned leave for 150 days =

Rs. 36,000. He was entitled to 40 days leave for every completed year of service. He got

50% of his pension commuted in lump sum w.e.f March 1, 2007 and received

Rs.1,20,000 as commuted pension. Abhishek contributes Rs. 900 per month to RPF to

which his employer contributes an equal amount. What is the amount of commuted

pension for Abhishek that will be exempted from income tax?

A) NIL

B) Rs. 40,000

C) Rs. 80,000

D) Rs. 1,20,000

Page 13: Retirement Mocktest

Financial Planning Academy

Section 3: 19 Questions (4 Marks each):

1) Ahmed’s current annual expenditure is Rs. 100000/-. He is 30 years old and expects to

retire at age 55. His annual expenses are estimated to rise by 6% p.a. and his life

expectancy is 75 years. His post retirement annual expenses are estimated to be 80% of

his pre-retirement expenses. What will be his expenses on the first year of his retirement?

A Rs. 3,47,425/-

B Rs. 3,43,350/-

C Rs. 4,29,187/-

D Rs. 4,02,350/-

2) A client has the need to provide for his child's college education costs. He envisages

that four annual payments of Rs 20,000/-, in current money terms, would be needed

beginning 15 years from now. Assuming level of inflation rate at 5% per annum and that

the fund earns 8% per annum returns throughout; calculate the present value to be placed

on this liability when carrying out a needs analysis for this client. (Round of your answer

to the nearest '000')

A Rs. 49,000/-

B Rs. 50,000/-

C Rs. 24,000/-

D Rs. 23,000/-

3) Mr. Nirav an affluent broker wants to retire at 45. He wants to maintain his present

living standard. He spends Rs.3,25,000/- a year. He is expected to live upto 85. Inflation

is to be assumed at 4% and expected returns are 7% p.a. How can he achieve this? He is

at present 30 years old. What is the nest egg required at age 45 and what amount shall he

save every year to meet his plan? His present investments are Rs.10,00,000/-

.(Assumption: All computations for interest, spend and savings compound annually,

assuming beginning of the period investment).

A Nest egg and savings required will be Rs.1,47,73,065/- and Rs.7,40,530/- respectively.

B Nest egg and savings required will be Rs.1,41,82,819/- and Rs.4,24,865/- respectively.

C Nest egg and savings required will be Rs.1, 27,73,065/- and Rs.4,38,300/- respectively.

D Nest egg and savings required will be Rs.2, 51,00,065/- and Rs.5, 41,093/-respectively.

4) Ms. Rekha is 45 years old and plans to retire at 50. Her life expectancy is 70 years.

Ms. Sushma, her Financial Planner, estimates that her client will require Rs.45,000/- in

the first month after retirement. Inflation rate is 4% p.a. and the rate of return is 6% p.a.

What will be the savings per year required in order to meet this?

A Rs. 15,90,000/- (Approx)

B Rs. 14,80,000/- (Approx)

C Rs. 16,90,000/- (Approx)

D Rs. 12,40,000/- (Approx)

Page 14: Retirement Mocktest

Financial Planning Academy

5) Vinita was recently divorced and has two children. The divorce decree requires that

she pay 1/3 of the college tuition cost for her children. The tuition cost is currently Rs.

15,000 per year and has been increasing at 7% per year. Her son and daughter are 12 and

16 respectively and will attend college for four years beginning at age 18. How much

should she save each month, beginning today for the next five years to finance education

for both the children (in nearest rupee)? Assume that her after-tax rate of return will be

9% and that general inflation has been 4% p.a.

A) Rs. 750

B) Rs. 745

C) Rs. 2,235

D) Rs. 2,500

6) Avinash pays his mortgage of Rs. 12 Lakh for 15 years at an interest rate of 7%.

Avinash makes the payments on a monthly basis. What is the total amount of interest

Avinash will pay over the term of the mortgage? (Select closest answer)

A) Rs. 6,47,000

B) Rs. 7,76,300

C) Rs. 7,30,200

D) Rs. 7,41,480

7) Nirav, aged 30, wants to retire at 45. He wants to maintain his present living standard.

He spends Rs. 3,25,000 a year. He is expected to live upto 85. Inflation is to be assumed

at 4% p.a and expected returns are 7% p.a. What is the nest egg required by Nirav at his

age 45 and what amount should he save every year end to meet his plan? His present

investments are Rs. 10,00,000.

A) Nest egg and savings required will be Rs. 1,41,82,828 and Rs. 4,54,606 respectively.

B) Nest egg and savings required will be Rs. 1,47,73,065 and Rs. 7,40,530 respectively.

C) Nest egg and savings required will be Rs. 1,27,73,065 and Rs. 4,38,300 respectively.

D) Nest egg and savings required will be Rs. 2,51,00,065 and Rs. 5, 41,093 respectively.

8) Laxman is an NRI who has been working in the US for the past 5 years. He is aged 40.

He has been saving Rs. 8 lakh per annum for the past 5 years and hopes to save the same

amount for the next 10 years that he plans to live in the US. He would like to return to

India 10 years from now. The inflation-adjusted monthly income requirement for

Laxman, as estimated by the planner, is Rs. 80,000 in the year in which he returns to

India. It is estimated that inflation would remain at an average of 3% p.a for the next 30

years. His life expectancy is placed at 70 years. However if the estimated spend per

month, for his family is Rs. 90,000 p.m., and the rate of inflation is 4% p.a, how long will

his savings last?

Assumption: His investments will earn a rate of interest of 6% pa. Compounding to be

done on annuity certain basis throughout the problem.

A) The funds will last until Laxman is 81 years old.

B) The funds will not be adequate to fund his estimated life span of 70 years.

C) The funds will last until Laxman is 72 years old.

D) The funds will exhaust before Laxman is 71 years old.

Page 15: Retirement Mocktest

Financial Planning Academy

9) Ajay has working in two companies viz. A Co. and B Co. He retires from A Co. on

November 30, 1988 (salary at the time of retirement Rs. 2,600 per month) and receives

Rs. 22,000 as gratuity out of which Rs. 20,000 was exempted. He also retires from B Co.

on December 10, 2006 after 28 years and 8 months of service and receives Rs. 2,90,000

as death-cum-retirement gratuity. His average basic salary drawn from B Co. for the

preceding 10 months ending on November 30, 2006 is Rs. 18,200 per month. Further he

receives Rs. 1,000 per month as DA, 80% of which forms a part of his retirement benefits

and 6% commission on turnover achieved by him. Total turnover achieved by him during

10 months ending November 30, 2006 is Rs.2,00,000. What is the gratuity exempted

from tax for Ajay for AY 2007-08? Assume neither of the companies A and B are

covered under payment of Gratuity Act 1972.

A) Rs. 2,80,000

B) Rs. 2,82,800

C) Rs. 3,30,000

D) Rs. 2,62,800

10) Girish received an inheritance of Rs. 2 Lakh. He wants to withdraw equal periodic

payments at the beginning of each month for 10 years starting after 10 years. He expects

to earn 12% annual interest, compounded monthly on his investments. How much can he

receive each month? .

A) Rs. 9,470

B) Rs. 9,376

C) Rs. 8,912

D) Rs. 8,824

11) The cash purchase price of an item is Rs. 2,00,000. The selling company however

offers installment plan, which allows an immediate payment of Rs. 10,000 and a series of

5 half-yearly payments thereafter. The first installment is payable after one and a half

year. If the company wants rate of interest of 10% P.A. compounded half-yearly, what

will be the half-yearly installment?

A) Rs. 43,885

B) Rs. 48,383

C) Rs. 50,802

D) Rs. 29,412

12) Sundar invests a sum of Rs. 72,000 at 5% p.a. After 7 years the rate of interest was

changed to 5% p.a, compounded half yearly. After a further period of 3 years the rate was

again changed to 6% p.a, compounded quarterly. What will Sundar get at the end of 15

years of commencement?

A) Rs. 1,40,000

B) Rs. 1,48,251

C) Rs. 1,58,242

D) Rs. 1,55,000

Page 16: Retirement Mocktest

Financial Planning Academy

13) Alok, age 25 years, plans to retire at age 60 and his life expectancy is 75 years. His

current expenditure is Rs. 2,00,000 annually. He estimates no reduction of expenses post

retirement. How much will he save per annum to achieve his target, if inflation rate is 6%

and expected yield from investment is 10%? Assume he wishes to leave an estate of 10%

of his savings at the time of retirement.

A) Rs. 73,878

B) Rs. 9,612

C) Rs. 66,490

D) Rs. 8,651

14) Ajay and Bela Mahera have two children ages 5 and 7. The Mehera.s want to start

saving for their children.s education. Each child will spend 6 years at college and will

begin at age 18. College currently costs Rs. 20000 per year and is expected to increase at

6% per year. Assuming the Mahera.s can earn an annual compound return of 12% and

inflation is 4%, how much must the Mehera.s deposit at the end of each year to pay for

their children.s educational requirements until the youngest is out of school? Assume that

educational expenses are withdrawn at the beginning of each year and that the last deposit

will be made at the beginning of the last year of the younger child.s college education.

A Rs. 15230

B Rs. 14,989

C Rs. 12,386

D None of the above

15) Mr. Nirav, an affluent broker, wants to retire at 45. He wants to maintain his present

living standard. He spends Rs. 3.25 lakh a year. He is expected to live upto 85. Inflation

is to be assumed at 4% and expected returns are 7% p.a. How can he achieve this? He is

at present 30 years old. What is the nest egg required at age 45 and what amount shall he

save every year to meet his plan? His present investments are Rs. 10 lakh.

1 Nest egg and savings required will be Rs. 13785155 and Rs. 438780 respectively.

2 Nest egg and savings required will be Rs. 14773065 and Rs. 740530 respectively.

3 Nest egg and savings required will be Rs. 12773065 and Rs. 438300 respectively.

4 Nest egg and savings required will be Rs. 25100065 and Rs. 541093 respectively.

16) Mr. Ramesh retired from PTC Ltd after completing service of 29 years and 9 months.

His salary at the time of retirement was Rs. 10500 per month while the average salary

drawn for the preceding 10 months worked out to Rs. 9800 per month. The actual amount

of gratuity received by him at the time of retirement was Rs. 3.25 lakh. Calculate the

amount of gratuity exempt from tax assuming that he is covered by the provisions of

Payment of Gratuity Act, 1972?

A Rs. 294000

B Rs. 181730

C Rs. 315000

D None of the above.

Page 17: Retirement Mocktest

Financial Planning Academy

17) Ms. Reshma is 35 years old and plans to retire at 50. Her life expectancy is 60 years.

Ms. Zarina her Financial Planner, estimates that her client will require Rs.65000 in the

first month after retirement. Inflation rate is 3% p.a. and the rate of return is 5% p.a. What

will be the savings per year required in order to meet this?

A Rs. 328300

B Rs. 345897

C Rs. 315894

D Rs. 320458

18) Prashant is an employee in the private sector to whom Payment of Gratuity Act does

not apply. He retires after eight years of service. Amongst other retirement benefits, he

also receives a gratuity of Rs. 2,50,000. His average salary for the past ten months, before

resigning, was Rs. 42,000 per month. He desires to know how much of the gratuity

amount, if any, is taxable?

A) Rs. 82,000

B) Nil

C) Rs. 2,50,000

D) Rs. 1,68,000

19) Nitish is a 30 year-old self employed youth and has been using the PPF account to

accumulate Rs. 30,000 per year, for his future needs. The PPF account provides a

compounded return of 8% p.a. He does not have a clear view yet on financial goals and

needs but has been saving as a habit, for the last 5 years. Nitish is willing to look at a

lifestyle after retirement that fits into a fixed Rs. 3,00,000 per annum spend, for an

estimated 15 years. What is the spending opportunity for Nitish, at the time of his

retirement at 60 years, given his saving and assuming a rate of 6% on his funds after

retirement? (Assumption: All computations for interest spend and savings compound

annually, assuming beginning of the period investment. Answers to be rounded off to the

nearest 5 rupees).

A) Rs. 20,81,345

B) Rs. 18,54,425

C) Rs. 14,26,890

D) Rs. 24,94,570